GMAT Club
close
Generic [Bot] online
My chats
Join group
New group
New private
Archived groups

GMAT Verbal Chat

GMAT Quantitative Chat

GMAT Data Insight Chat

MBA Chat

go back
Quant Question of the Day 6 members online out of 124274
dots
Forum discussion
Chat members
User avatar
guys can anyone make a separate chat for GRE
chevron down
Quote
Private
Delete
Pin
User avatar
y=(x^2-x)(x-2)=x(x-1)(x-2). So, y will be 0 when x=0,1, and 2, that is the curve will cut graph at 0, 1 and 2. Only C and D left........... Next, when x>2, all three x, x-1 and x-2 will be positive, and product of three positive is positive. Thus after cutting x-axis at 2, the line should give y as positive always..... OR You could just take x as 3, and see that y=3*1*2=6. D gives you positive value while C gives a negative value for y at x=3...answer is D
chevron down
Quote
Private
Delete
Pin
User avatar
for rest GMAT students, Alex these questions are confusing and not important
chevron down
Quote
Private
Delete
Pin
User avatar
aayushimehta12 is correct. The questions could be posted in GRE forum.
chevron down
Quote
Private
Delete
Pin
User avatar
Thank you, because the entire time I am looking at questions and being confused is this are and gmat, am I supposed to know the answer or not
chevron down
Quote
Private
Delete
Pin
User avatar
so Nitish and Alex, requesting you to please post these questions elsewhere
chevron down
Quote
Private
Delete
Pin
User avatar
Just opened a new chat on GRE. Please post Quant questions/queries here https://gmatclub.com/forum/mchat.php?ro ... t73WUjV8wf
chevron down
Quote
Private
Delete
Pin
User avatar
Sorry for that actually my xm is near
chevron down
Quote
Private
Delete
Pin
User avatar
HarshR9 wrote:
My attempt. Hope this is correct.

i am also getting 91
chevron down
Quote
Private
Delete
Pin
User avatar
chevron down
Quote
Private
Delete
Pin
User avatar
:heart
chevron down
Quote
Private
Delete
Pin
User avatar
One more new GMAT Club Tests question on Absolute Values:

https://gmatclub.com/forum/if-a-b-b-c-2 ... 28843.html
chevron down
Quote
Private
Delete
Pin
User avatar
Thank you Alex
chevron down
Quote
Private
Delete
Pin
User avatar
chevron down
Quote
Private
Delete
Pin
User avatar
chetan2u, Just for your information that nobody is there to respond in GRE sub-forum or the GRE prep club. I have checked myself. So, I request you to pls allow GRE students to post their queries here itself till the time they won’t get any help from the Gre Sub-forum. So that they won’t get lost. Please consider my request.
chevron down
Quote
Private
Delete
Pin
User avatar
I am really confused here, isn’t both 2 & 6 are the correct answer ?

Screenshot 2024-04-22 at 3.16.38 AM.png

chevron down
Quote
Private
Delete
Pin
User avatar
Rangerwarrior wrote:
I am really confused here, isn’t both 2 & 6 are the correct answer ?

Rangerwarrior, you don’t require to solve the equation to get the answer. On right side you have a square root,which cannot be NEGATIVE. So, left side also cannot be negative. But x as 2 gives 2-5 or -3. Similarly from x-5 >=0 means x can be 5 or 6. x as 5 will give 0 on the left side but the right side will not be 0(you can check the unts digit only). Answer x=6
chevron down
Quote
Private
Delete
Pin
User avatar
poojaarora1818, sure put your question here as we are here to help you all. But do post it in the new chat opened for the purpose. It will become active only when we start shifting our questions there. https://gmatclub.com/forum/mchat.php?room_id=50050793
chevron down
Quote
Private
Delete
Pin
User avatar
chetan2u wrote:
poojaarora1818, sure put your question here as we are here to help you all. But do post it in the new chat opened for the purpose. It will become active only when we start shifting our questions there. https://gmatclub.com/forum/mchat.php?room_id=50050793

chetan2u, Thank you for your positive response. But let me tell you that I am not a GRE student. I am a Gmat student. I am doing this because I can understand the pain of a student who really struggles when they seek replies for their respective quieries. That’s it. Thank you!
chevron down
Quote
Private
Delete
Pin
User avatar
Data Sufficiency Butler: April 2024
April 22DS 1DS 2
chevron down
Quote
Private
Delete
Pin
User avatar
Problem Solving Butler: April 2023
April 22PS 1PS 2
chevron down
Quote
Private
Delete
Pin
User avatar
Howdy folks! Hope everyone’s prep is going fine :) I wanted to know if there’s a more efficient way of solving below without listing all the terms (TIA!): Sn is a sequence such that it’s nth term is sum of first npositive integers. Tn is a sequence such that its first 2 terms are 1 and starting from the 3rd term, each term is the sum of the previous 2 terms. HOw many numbers less than 100 belong to both Sn and Tn ?
chevron down
Quote
Private
Delete
Pin
User avatar
If this question is already discussed somewhere can someone share link pls ? I searched online but found questions where first 3 terms or 6th/nth term is asked..
chevron down
Quote
Private
Delete
Pin
User avatar
poojaarora1818 wrote:
chetan2u, Thank you for your positive response. But let me tell you that I am not a GRE student. I am a Gmat student. I am doing this because I can understand the pain of a student who really struggles when they seek replies for their respective quieries. That’s it. Thank you!

poojaarora1818, it is good to see you thinking of other students too.
chevron down
Quote
Private
Delete
Pin
User avatar
Rucha.Shukla wrote:
Howdy folks! Hope everyone’s prep is going fine :) I wanted to know if there’s a more efficient way of solving below without listing all the terms (TIA!): Sn is a sequence such that it’s nth term is sum of first npositive integers. Tn is a sequence such that its first 2 terms are 1 and starting from the 3rd term, each term is the sum of the previous 2 terms. HOw many numbers less than 100 belong to both Sn and Tn ?

Could you please post entire question with the OA and the options exactly as seen in the original source? Also, could you please tell the source? Thank you!
chevron down
Quote
Private
Delete
Pin
User avatar
Rucha.Shukla wrote:
Howdy folks! Hope everyone’s prep is going fine :) I wanted to know if there’s a more efficient way of solving below without listing all the terms (TIA!): Sn is a sequence such that it’s nth term is sum of first npositive integers. Tn is a sequence such that its first 2 terms are 1 and starting from the 3rd term, each term is the sum of the previous 2 terms. HOw many numbers less than 100 belong to both Sn and Tn ?

I don’t think there would be a better approach. Even if there’s, that would be complicated and time consuming. Below 100, there is 14 terms in series Tn. So relatively easy
chevron down
Quote
Private
Delete
Pin
User avatar
Tn is a fibbonnaci series, so there would be 11 terms less than 100
But the question is asking how many terms would belong in both Tn and Sn, which would be 4 terms (1,3,21,55)
chevron down
Quote
Private
Delete
Pin
User avatar
Rucha.Shukla wrote:
Howdy folks! Hope everyone’s prep is going fine :) I wanted to know if there’s a more efficient way of solving below without listing all the terms (TIA!): Sn is a sequence such that it’s nth term is sum of first npositive integers. Tn is a sequence such that its first 2 terms are 1 and starting from the 3rd term, each term is the sum of the previous 2 terms. HOw many numbers less than 100 belong to both Sn and Tn ?

May not be the most elegant but a faster method could be knowing what Tn is looking for. It is talking of sum of first n positive integers, so nth term is n(n+1)/2, that is it is product of two consecutive numbers divided by 2. So, scan the Sn for these type of numbers. Sn = 1,1,2,3,5,8,13,21,34,55,89.........1*2/2 =1 , so 1...2*3/2=3, so 3....6*7/2=21, so 21.....7*8/2=28, not there...10*11/2=55, so 55
chevron down
Quote
Private
Delete
Pin
User avatar
On a radio call-in show, the 6th caller and every 6th caller thereafter were given a T-shirt. Also, the 15th caller and every 15th caller thereafter were given movie tickets. A total of 78 people called the show, all at different times. How many of the callers were given neither a T-shirt nor movie tickets?
___ callers
chevron down
Quote
Private
Delete
Pin
User avatar
Last week, each employee at a small company, except for one employee, made a donation to a certain charity. The average (arithmetic mean) amount of the donations was $39 per person. This week, after the remaining employee made a donation, the average amount of the donations increased to $40 per person. If the remaining employee’ s donation was between $54 and $61, which of the following could be the total number of employees at the company?

Indicate all such numbers.
14,17,20,23,26
chevron down
Quote
Private
Delete
Pin
User avatar
Does anyone else want to try this question?
https://gmatclub.com/forum/two-bicycle- ... 28866.html
chevron down
Quote
Private
Delete
Pin
User avatar
Data Sufficiency Butler: April 2024
April 23DS 1DS 2
ยญ
chevron down
Quote
Private
Delete
Pin
User avatar
April 23
PS 1
PS 2
chevron down
Quote
Private
Delete
Pin
User avatar
I think it’s C we can’t find exact time for 6 with A cuz it’s ratio together we can find time
chevron down
Quote
Private
Delete
Pin
User avatar
1rotation/xmin 1rotation/3xmin -> 40rotation/1min so we can get x =1/30 so A 10rotation/1min B 30rotation/1min 6rotation/20rotation/1min = 3/10min -> 18seconds
chevron down
Quote
Private
Delete
Pin
User avatar
@Bunuel is it C?
chevron down
Quote
Private
Delete
Pin
User avatar
Oh mistake average so x -> 1/60 min 1 seconds --> 9 seconds
chevron down
Quote
Private
Delete
Pin
User avatar
Can I get help please

Quant 4.png

Quant 3.png

Quant 2.png

Quant 5.png

Quant 1.png

chevron down
Quote
Private
Delete
Pin
User avatar
Check the links for detailed solutions of these questions below.
1st question: https://gmatclub.com/forum/at-a-recent- ... 16434.html
chevron down
Quote
Private
Delete
Pin
User avatar
chevron down
Quote
Private
Delete
Pin
User avatar
chevron down
Quote
Private
Delete
Pin
User avatar
chevron down
Quote
Private
Delete
Pin
User avatar
chevron down
Quote
Private
Delete
Pin
User avatar
Try searching a question in the respective forum before posting in the future. That could save you time.
chevron down
Quote
Private
Delete
Pin
User avatar
Hope this helps.
chevron down
Quote
Private
Delete
Pin
User avatar
Can someone please help me with this question? https://gmatclub.com/forum/two-bicycle- ... l#p3387223
chevron down
Quote
Private
Delete
Pin
User avatar
Not sure if the given solutions are correct
chevron down
Quote
Private
Delete
Pin
User avatar
akanksha14_11 wrote:
Can someone please help me with this question? https://gmatclub.com/forum/two-bicycle- ... l#p3387223

I think, it is option C only.
Take choice A, it states that wheel B rotates 3times faster than wheel A.
With choice A alone, we cannot come to the conclusion in how many seconds wheel B completes 6 times more rotations than wheel A.
So eliminate A & D options.

Take choice B, it states that average revolution of both the wheels are 40 / minute.

So , average revolution of wheel (A + B)/2 = 40
So max possible is A+B = 80 revolutions / minute.
This is also not helpful

But if we substitute choice A , i.e Wheel B has rate of rotation of 3 times than wheel B
Wheel A’s rate of rotation = B/3

Sub, that rate in Choice B ,

A+B = 80
(B/3)+B=80
B+3B=240
B=60 revolutions per minute
Therefore, Wheel A’s rotation is 20 rotations per minute.

We can therefore deduce that after 9 seconds, Wheel A would have rotated 3 times whereas 9 revolution by wheel B, therefore there’s a difference of 6 revolutions between both the wheel.
chevron down
Quote
Private
Delete
Pin
User avatar
So option C
chevron down
Quote
Private
Delete
Pin
User avatar
akanksha14_11, Hi!
What made you feel confused regarding this question?

https://gmatclub.com/forum/two-bicycle- ... l#p3387223 ?
chevron down
Quote
Private
Delete
Pin
User avatar
I don’t think option A is alone , sufficient
chevron down
Quote
Private
Delete
Pin
User avatar
Because they are asking specifically in numerics
chevron down
Quote
Private
Delete
Pin
User avatar
Option C
chevron down
Quote
Private
Delete
Pin
User avatar
can someone helo in this please? am getting 1/6

Capture.zdasdPNG.PNG

chevron down
Quote
Private
Delete
Pin
User avatar
Source please?
chevron down
Quote
Private
Delete
Pin
User avatar
practice exam 4
chevron down
Quote
Private
Delete
Pin
User avatar
Red =1330
Blue=2660
P(blue)=2660/8000
chevron down
Quote
Private
Delete
Pin
User avatar
Approximately 1/3
chevron down
Quote
Private
Delete
Pin
User avatar
twice as many blue as red means red=2blue or blue=2 red
chevron down
Quote
Private
Delete
Pin
User avatar
B=2r
chevron down
Quote
Private
Delete
Pin
User avatar
Blue= 2 Red
chevron down
Quote
Private
Delete
Pin
User avatar
A lottery box contains 8,000 tickets, each of which is red or blue or green: b + g + r = 8000
The box contains twice as many blue tickets as red tickets ยญb = 2r
The number of green tickets is 20 more than the number of red and blue tickets combined: g - 20 = b + r

The question ask to find the approximate value of b/8,000.

Substituting g - 20 = b + r into b + g + r = 8000 we get g + (g - 20) = 8000, which gives g = 4,010.
Substituting g = 4,010 into g - 20 = b + r we get 4,010 - 20 = b + r, which gives r = 3,990 - b.
Substituting r = 3,990 - b into b = 2r we get b = 2(3,990 - b), which gives b = 2,660.

Therefore, b/8,000 = 2,660/8,000 = 133/400 โ‰ˆ 1/3.

Answer: A.ยญ
chevron down
Quote
Private
Delete
Pin
User avatar
chevron down
Quote
Private
Delete
Pin
User avatar
can someone help me pls, can’t figure out the mistake I am making

3.JPG

chevron down
Quote
Private
Delete
Pin
User avatar
Take 100 employees at this company. 60 participate in the retirement plan. 20% of this 60 also participate in the health plan. So 12 people participate in the retirement and health plan.

48 people in total signed up for health plan. Out of them, 12 also participate in retirement. So 12/48 -> 1/4
chevron down
Quote
Private
Delete
Pin
User avatar
Bunuel, can you please provide solution for this question ?

https://gmatclub.com/forum/a-share-of-c ... 28317.html
chevron down
Quote
Private
Delete
Pin
User avatar
is it because the "trading volume" is given in A that the answer is B and not D, since trading volume is always less than total volume of a company’s shares ?
chevron down
Quote
Private
Delete
Pin
User avatar
Of the employees at a certain company, 60 percent participate in the company’s retirement plan; and of those employees, 20 percent also participate in the company’s health plan. If 48 percent of the company’s employees participate in its health plan, what fraction of those employees also participate in the company’s retirement plan?

A. 1/4
B. 1/3
C. 1/2
D. 2/3
E. 3/4ยญ


20% of 60%, thus 12% of the employees participate both in the retirement plan and health plan. The question asks what fraction of the employees participating in the health plan, participate in both plans. Since we got that 12% participate in both, then the answer is both/health = 12/48 = 1/4.

Answer: A.
chevron down
Quote
Private
Delete
Pin
User avatar
divyansh843, is this a GMAT Prep Focus question? Which test? Thank you!
chevron down
Quote
Private
Delete
Pin
User avatar
Data Sufficiency Butler: April 2024
April 24DS 1DS 2
chevron down
Quote
Private
Delete
Pin
User avatar
Problem Solving Butler: April 2023
April 24PS 1PS 2

ยญ
chevron down
Quote
Private
Delete
Pin
User avatar
is this an OG question?
chevron down
Quote
Private
Delete
Pin
User avatar
Data Sufficiency Butler: April 2024
April 25DS 1DS 2
ยญ
chevron down
Quote
Private
Delete
Pin
User avatar
Problem Solving Butler: April 2023
April 25PS 1PS 2
chevron down
Quote
Private
Delete
Pin
User avatar
Bunuel do you have the link for
chevron down
Quote
Private
Delete
Pin
User avatar
all butler questions ?
chevron down
Quote
Private
Delete
Pin
User avatar
Baye’s theorem is all you need
chevron down
Quote
Private
Delete
Pin
User avatar
Data Sufficiency Butler: April 2024
April 26DS 1DS 2
chevron down
Quote
Private
Delete
Pin
User avatar
Problem Solving Butler: April 2023
April 26PS 1PS 2
chevron down
Quote
Private
Delete
Pin
User avatar
chevron down
Quote
Private
Delete
Pin
User avatar
https://gmatclub.com/forum/for-integers ... 29065.html

Can anyone help me with this question?
chevron down
Quote
Private
Delete
Pin
User avatar
devashish2407 wrote:
https://gmatclub.com/forum/for-integers ... 29065.html Can anyone help me with this question?

I think the answer should be A, however they say that’s incorrect.
chevron down
Quote
Private
Delete
Pin
User avatar
b and d both are integers so both can only be equal to 1 or -1 if bd=1 so c=a, sufficient. However, b+d=3 is insufficient , we can check this with examples , b = 4 d =-1 , b+d =3 and b>d so c>a. But if b=1 and d=2, b+d = 3 but b<d so c<a. Please let me know if I missed something.
chevron down
Quote
Private
Delete
Pin
User avatar
devashish2407 wrote:
https://gmatclub.com/forum/for-integers ... 29065.html Can anyone help me with this question?

I think the answer should be E as both statement together doesnot give any concrete information about the relation between a and c even if we substitute the value of the respective numbers b and d

from (i) we get b = 1/d
or (b^2) = c/a (so we are not sure about the value of c/a which could be a fraction or a whole number (NOT sufficient)
from. (ii) we get b = 3-d
on substituting we get c/a = d/(3-d) which also doesn’t give a concrete value as the value of d is decisive (NOT sufficient)

On combining both we cannot find any exact value of d which could suffice a single condition

therefore both together not sufficient (E) should be the answer
chevron down
Quote
Private
Delete
Pin
User avatar
but the question mentions that it is an integer
chevron down
Quote
Private
Delete
Pin
User avatar
so it cannot be a fraction
chevron down
Quote
Private
Delete
Pin
User avatar
The answer to above question is E only as correctly described by u guys.
chevron down
Quote
Private
Delete
Pin
User avatar
I have a generic question. Is there any relationship between AM, GM, HM, mean and median?

U can decide the case/ situation and illustrate your opinion.
chevron down
Quote
Private
Delete
Pin
User avatar
Yes, we have many questions on relationships between mean and median
chevron down
Quote
Private
Delete
Pin
User avatar
and mode even sometimes
chevron down
Quote
Private
Delete
Pin
User avatar
depending on question, any kind of relationship can be drawn out, but I have attempted multiple questions with mean, median and mode, specially in D.S
chevron down
Quote
Private
Delete
Pin
User avatar
That’s great ๐Ÿ‘. However, my question was on broader aspect.
Is there any relationship between AM GM HM mean and median
chevron down
Quote
Private
Delete
Pin
User avatar
AM means arithmetic mean, G is geometric mean etc
chevron down
Quote
Private
Delete
Pin
User avatar
Harmonic Progression is rarely ever tested on GMAT, atleast I’ve not seen in over 2 years
chevron down
Quote
Private
Delete
Pin
User avatar
One relation that comes to mind is, "If numbers are in AP, then Mean = Median"
chevron down
Quote
Private
Delete
Pin
User avatar
That is the one I was talking about too
chevron down
Quote
Private
Delete
Pin
User avatar
I stand corrected. GMAT focus tests harmonic mean - above is an example from GMAT prep.
chevron down
Quote
Private
Delete
Pin
User avatar
As was pointed out, I also have not seen it brought up much in the past.
chevron down
Quote
Private
Delete
Pin